Communities

Writing
Writing
Codidact Meta
Codidact Meta
The Great Outdoors
The Great Outdoors
Photography & Video
Photography & Video
Scientific Speculation
Scientific Speculation
Cooking
Cooking
Electrical Engineering
Electrical Engineering
Judaism
Judaism
Languages & Linguistics
Languages & Linguistics
Software Development
Software Development
Mathematics
Mathematics
Christianity
Christianity
Code Golf
Code Golf
Music
Music
Physics
Physics
Linux Systems
Linux Systems
Power Users
Power Users
Tabletop RPGs
Tabletop RPGs
Community Proposals
Community Proposals
tag:snake search within a tag
answers:0 unanswered questions
user:xxxx search by author id
score:0.5 posts with 0.5+ score
"snake oil" exact phrase
votes:4 posts with 4+ votes
created:<1w created < 1 week ago
post_type:xxxx type of post
Search help
Notifications
Mark all as read See all your notifications »
Q&A

Post History

#1: Initial revision by user avatar TheCodidacter, or rather ACodidacter‭ · 2024-04-05T17:29:32Z (about 1 month ago)
Prove that $\forall x\in\Bbb R:\lfloor x^2\rfloor-\lfloor rx\rfloor\ge-1\iff|r|\le2$.
> Prove that the inequality $\lfloor x^2\rfloor-\lfloor rx\rfloor\ge-1$ holds true for all real numbers $x$ if and only if $|r|\le2$.

My (proposed) solution, which took me around 2 hours:
> First, we reframe the inequality to $\lfloor x^2\rfloor-\lfloor rx\rfloor+1\ge0$.Suppose $r=a$ satisfies the inequality for all real numbers $x$. Consequently, it also does for all real numbers $-x$:
$$\lfloor(-x)^2\rfloor-\lfloor a(-x)\rfloor+1\ge0$$
$$\lfloor x^2\rfloor-\lfloor (-a)x\rfloor+1\ge0$$
And we get that $r=-a$ satisfies the condition if and only if $r=a$ does too. Then, we only need to consider cases where $r\ge0$.

> Furthermore, let us consider cases where $r\gt2$. The inequality holds for all $x$, so we test it for $0\lt x=\frac2r\lt1$.
$$\lfloor x^2\rfloor-\lfloor2\rfloor+1\ge0$$
$$0-2+1\ge0$$, contradiction. We are left with $r\in[0,2]$.

> Another observation. If $a\ge0$ and $r=a$ fulfills the condition,
$$\lfloor x^2\rfloor-\lfloor rx\rfloor+1\ge0$$
For $x\ge0$, we find that $r\lt a$ also fulfills the condition. All three terms are positive when $x\lt0$, and therefore for $a\ge0$, all $r\lt a$ fulfill the condition if $r=a$ does. Therefore, all that's left to do is to prove the case for $r=2$, and we are done.

> Notice that $\lfloor a+b\rfloor-1\le\lfloor a\rfloor+\lfloor b\rfloor$, and $(x-1)^2\ge0\implies\lfloor x^2-2x+1\rfloor=\lfloor x^2-2x\rfloor+1\ge0$.
Combining the two,
$$0\le\lfloor x^2-2x\rfloor-1+2\le\lfloor x^2\rfloor+\lfloor-2x\rfloor+2$$.
And as long as $a\notin\Bbb Z$,
$$\lfloor-a\rfloor=-\lfloor a\rfloor-1$$. Applying that, for $2x\notin\Bbb Z$,
$$0\le\lfloor x^2\rfloor+\lfloor-2x\rfloor+2=\lfloor x^2\rfloor-\lfloor2x\rfloor+1\ \blacksquare$$
And if $n=2x\in\Bbb Z$,
$$0\le\left\lfloor\frac14n^2\right\rfloor-n+2$$.
Notice that $n\equiv1\pmod4$ or $n\equiv3\pmod4\iff n^2\equiv1\pmod4$. So,
$$0\le\frac14(n^2-1)-n+2$$
$$0\le\frac14n^2-n+\frac74$$
$$0\le\left(\frac12n-1\right)^2+\frac34\ \blacksquare$$.

Is my proposed solution correct? And better yet, is there a faster (and shorter) way to find this proof?

Thank you in advance, and feel free to correct any errors I made!